Given prime numbers $p,q$ and $r$, $p|qr − 1$, $q|rp − 1$, and $r|pq − 1$. What is the value of all possible $pqr$?What is the value of $Sigma frac1pqr$ with $frac1a>frac1pqr$?Can one show a beginning student how to use the $p$-adics to solve a problem?Evaluate $lim_n to inftyBigl[bigl(prod_k=1^nfrac2k2k-1bigr)bigl(int_-1^inftyfrac(cosx)^2n2^x,dxbigr)Bigr]$Mathematical Reflections J339: Solving $fracx-1y+1 + fracy-1z+1 + fracz-1x+1 = 1$ in positive integersProof verification - working with integersFind all pair of $(x,y)$ such that $fracx^2+y^2x-y$ is an integer and divides 1995.Find all pairs of prime numbers $p$ and $q$ such that $,p^2-p-1=q^3.$Simple solution to Question 6 from the 1988 Math OlympiadNumber Theory : Prime Divisor Property ProofShowing that $a^2+b^2+c^2+d^2+e^2+65=abcde$ has integer solutions greater than $2018$?

How can bays and straits be determined in a procedurally generated map?

Is there a familial term for apples and pears?

Do airline pilots ever risk not hearing communication directed to them specifically, from traffic controllers?

My colleague's body is amazing

Circuitry of TV splitters

"The augmented fourth (A4) and the diminished fifth (d5) are the only augmented and diminished intervals that appear in diatonic scales"

How is the relation "the smallest element is the same" reflexive?

Is it legal to have the "// (c) 2019 John Smith" header in all files when there are hundreds of contributors?

Can you lasso down a wizard who is using the Levitate spell?

Why is "Reports" in sentence down without "The"

Need help identifying/translating a plaque in Tangier, Morocco

Is it possible to make sharp wind that can cut stuff from afar?

Could a US political party gain complete control over the government by removing checks & balances?

How is it possible for user's password to be changed after storage was encrypted? (on OS X, Android)

Simulate Bitwise Cyclic Tag

how to create a data type and make it available in all Databases?

Can Medicine checks be used, with decent rolls, to completely mitigate the risk of death from ongoing damage?

Is there really no realistic way for a skeleton monster to move around without magic?

What do you call something that goes against the spirit of the law, but is legal when interpreting the law to the letter?

The use of multiple foreign keys on same column in SQL Server

XeLaTeX and pdfLaTeX ignore hyphenation

How do we improve the relationship with a client software team that performs poorly and is becoming less collaborative?

Why is an old chain unsafe?

How to use Pandas to get the count of every combination inclusive



Given prime numbers $p,q$ and $r$, $p|qr − 1$, $q|rp − 1$, and $r|pq − 1$. What is the value of all possible $pqr$?


What is the value of $Sigma frac1pqr$ with $frac1a>frac1pqr$?Can one show a beginning student how to use the $p$-adics to solve a problem?Evaluate $lim_n to inftyBigl[bigl(prod_k=1^nfrac2k2k-1bigr)bigl(int_-1^inftyfrac(cosx)^2n2^x,dxbigr)Bigr]$Mathematical Reflections J339: Solving $fracx-1y+1 + fracy-1z+1 + fracz-1x+1 = 1$ in positive integersProof verification - working with integersFind all pair of $(x,y)$ such that $fracx^2+y^2x-y$ is an integer and divides 1995.Find all pairs of prime numbers $p$ and $q$ such that $,p^2-p-1=q^3.$Simple solution to Question 6 from the 1988 Math OlympiadNumber Theory : Prime Divisor Property ProofShowing that $a^2+b^2+c^2+d^2+e^2+65=abcde$ has integer solutions greater than $2018$?













3












$begingroup$


Let $p,q$ and $r$ be prime numbers. It is given that $p$ divides $qr − 1$, $q$ divides $rp − 1$, and $r$ divides $pq − 1$.
Determine all possible values of $pqr$.



I think I'm missing something in this problem. There's no solutions online so I was wondering if you guys could show me what I'm doing wrong.



We must have one of $p,q,r = 2$ as $pq - 1$, $pr - 1$, $qr - 1$ are congruent to $0 mod 2$ if $p,q,r > 2$, so wlog let $p = 2$ (more than one of $p,q,r$ being 2 gives contradiction). Then $q$ divides $2r - 1$ (so $2r - 1 = xq$) and $r$ divides $2q - 1$ (so $2q - 1 = yr$). Solving for r we have that $(xq + 1)/2 = (2q - 1)/y$ hence $ q = (2 + y)/(4 - xy)$. This gives the only pair as $(3,5)$, so $pqr$ can only be $30$?



The way the question is phrased leads me to believe there's more to it, and also it's the last question on an olympiad so it should take longer than 5 minutes.



Re-did it, is this okay?



If we write $ rq - 1 = px $ and $ rp - 1 = qy$ we obtain $ p = fracr + yr^2 - xy $ and $ q = fracr + xr^2 - xy $ (and clearly we can't have $ r^2 = xy $) so $q(r + y) = p(r + x)$. Then subbing $ x = frac3q - 1p $ and $y = frac3p - 1q$ we obtain $r=3$ if $p$ doesn't equal $q$ (if $p=q$ then no solutions). Then it's just a matter of checking cases and we just get $(p,q,r) = (5,7,3), (2,5,3)$. But $(5,7,3)$ doesn't satisfy the fact that $r$ divides $pq - 1$ hence the only solution is $pqr = 30$. I hope that's ok










share|cite|improve this question











$endgroup$











  • $begingroup$
    Here's a MathJax tutorial :)
    $endgroup$
    – Shaun
    Jul 20 '14 at 18:21










  • $begingroup$
    Are negative primes allowed?
    $endgroup$
    – Mathmo123
    Jul 20 '14 at 18:23










  • $begingroup$
    Isn't a prime number positive by definition? Thanks Shaun I tried to make it better
    $endgroup$
    – user121591
    Jul 20 '14 at 18:25







  • 1




    $begingroup$
    I don't think your first line is correct. Suppose they are all greater than two. Then indeed all of the products of the form $pq-1$ are even. But that isn't a problem (even numbers can have odd prime divisors), and it doesn't immediately imply anything interesting.
    $endgroup$
    – Potato
    Jul 20 '14 at 18:35











  • $begingroup$
    By the way, I think you can piece a solution together from the comments here: nrich.maths.org/discus/messages/153904/149403.html?1281107322
    $endgroup$
    – Potato
    Jul 20 '14 at 18:40















3












$begingroup$


Let $p,q$ and $r$ be prime numbers. It is given that $p$ divides $qr − 1$, $q$ divides $rp − 1$, and $r$ divides $pq − 1$.
Determine all possible values of $pqr$.



I think I'm missing something in this problem. There's no solutions online so I was wondering if you guys could show me what I'm doing wrong.



We must have one of $p,q,r = 2$ as $pq - 1$, $pr - 1$, $qr - 1$ are congruent to $0 mod 2$ if $p,q,r > 2$, so wlog let $p = 2$ (more than one of $p,q,r$ being 2 gives contradiction). Then $q$ divides $2r - 1$ (so $2r - 1 = xq$) and $r$ divides $2q - 1$ (so $2q - 1 = yr$). Solving for r we have that $(xq + 1)/2 = (2q - 1)/y$ hence $ q = (2 + y)/(4 - xy)$. This gives the only pair as $(3,5)$, so $pqr$ can only be $30$?



The way the question is phrased leads me to believe there's more to it, and also it's the last question on an olympiad so it should take longer than 5 minutes.



Re-did it, is this okay?



If we write $ rq - 1 = px $ and $ rp - 1 = qy$ we obtain $ p = fracr + yr^2 - xy $ and $ q = fracr + xr^2 - xy $ (and clearly we can't have $ r^2 = xy $) so $q(r + y) = p(r + x)$. Then subbing $ x = frac3q - 1p $ and $y = frac3p - 1q$ we obtain $r=3$ if $p$ doesn't equal $q$ (if $p=q$ then no solutions). Then it's just a matter of checking cases and we just get $(p,q,r) = (5,7,3), (2,5,3)$. But $(5,7,3)$ doesn't satisfy the fact that $r$ divides $pq - 1$ hence the only solution is $pqr = 30$. I hope that's ok










share|cite|improve this question











$endgroup$











  • $begingroup$
    Here's a MathJax tutorial :)
    $endgroup$
    – Shaun
    Jul 20 '14 at 18:21










  • $begingroup$
    Are negative primes allowed?
    $endgroup$
    – Mathmo123
    Jul 20 '14 at 18:23










  • $begingroup$
    Isn't a prime number positive by definition? Thanks Shaun I tried to make it better
    $endgroup$
    – user121591
    Jul 20 '14 at 18:25







  • 1




    $begingroup$
    I don't think your first line is correct. Suppose they are all greater than two. Then indeed all of the products of the form $pq-1$ are even. But that isn't a problem (even numbers can have odd prime divisors), and it doesn't immediately imply anything interesting.
    $endgroup$
    – Potato
    Jul 20 '14 at 18:35











  • $begingroup$
    By the way, I think you can piece a solution together from the comments here: nrich.maths.org/discus/messages/153904/149403.html?1281107322
    $endgroup$
    – Potato
    Jul 20 '14 at 18:40













3












3








3


0



$begingroup$


Let $p,q$ and $r$ be prime numbers. It is given that $p$ divides $qr − 1$, $q$ divides $rp − 1$, and $r$ divides $pq − 1$.
Determine all possible values of $pqr$.



I think I'm missing something in this problem. There's no solutions online so I was wondering if you guys could show me what I'm doing wrong.



We must have one of $p,q,r = 2$ as $pq - 1$, $pr - 1$, $qr - 1$ are congruent to $0 mod 2$ if $p,q,r > 2$, so wlog let $p = 2$ (more than one of $p,q,r$ being 2 gives contradiction). Then $q$ divides $2r - 1$ (so $2r - 1 = xq$) and $r$ divides $2q - 1$ (so $2q - 1 = yr$). Solving for r we have that $(xq + 1)/2 = (2q - 1)/y$ hence $ q = (2 + y)/(4 - xy)$. This gives the only pair as $(3,5)$, so $pqr$ can only be $30$?



The way the question is phrased leads me to believe there's more to it, and also it's the last question on an olympiad so it should take longer than 5 minutes.



Re-did it, is this okay?



If we write $ rq - 1 = px $ and $ rp - 1 = qy$ we obtain $ p = fracr + yr^2 - xy $ and $ q = fracr + xr^2 - xy $ (and clearly we can't have $ r^2 = xy $) so $q(r + y) = p(r + x)$. Then subbing $ x = frac3q - 1p $ and $y = frac3p - 1q$ we obtain $r=3$ if $p$ doesn't equal $q$ (if $p=q$ then no solutions). Then it's just a matter of checking cases and we just get $(p,q,r) = (5,7,3), (2,5,3)$. But $(5,7,3)$ doesn't satisfy the fact that $r$ divides $pq - 1$ hence the only solution is $pqr = 30$. I hope that's ok










share|cite|improve this question











$endgroup$




Let $p,q$ and $r$ be prime numbers. It is given that $p$ divides $qr − 1$, $q$ divides $rp − 1$, and $r$ divides $pq − 1$.
Determine all possible values of $pqr$.



I think I'm missing something in this problem. There's no solutions online so I was wondering if you guys could show me what I'm doing wrong.



We must have one of $p,q,r = 2$ as $pq - 1$, $pr - 1$, $qr - 1$ are congruent to $0 mod 2$ if $p,q,r > 2$, so wlog let $p = 2$ (more than one of $p,q,r$ being 2 gives contradiction). Then $q$ divides $2r - 1$ (so $2r - 1 = xq$) and $r$ divides $2q - 1$ (so $2q - 1 = yr$). Solving for r we have that $(xq + 1)/2 = (2q - 1)/y$ hence $ q = (2 + y)/(4 - xy)$. This gives the only pair as $(3,5)$, so $pqr$ can only be $30$?



The way the question is phrased leads me to believe there's more to it, and also it's the last question on an olympiad so it should take longer than 5 minutes.



Re-did it, is this okay?



If we write $ rq - 1 = px $ and $ rp - 1 = qy$ we obtain $ p = fracr + yr^2 - xy $ and $ q = fracr + xr^2 - xy $ (and clearly we can't have $ r^2 = xy $) so $q(r + y) = p(r + x)$. Then subbing $ x = frac3q - 1p $ and $y = frac3p - 1q$ we obtain $r=3$ if $p$ doesn't equal $q$ (if $p=q$ then no solutions). Then it's just a matter of checking cases and we just get $(p,q,r) = (5,7,3), (2,5,3)$. But $(5,7,3)$ doesn't satisfy the fact that $r$ divides $pq - 1$ hence the only solution is $pqr = 30$. I hope that's ok







number-theory contest-math






share|cite|improve this question















share|cite|improve this question













share|cite|improve this question




share|cite|improve this question








edited Oct 11 '16 at 6:29









Irregular User

2,88251843




2,88251843










asked Jul 20 '14 at 18:18









user121591user121591

737




737











  • $begingroup$
    Here's a MathJax tutorial :)
    $endgroup$
    – Shaun
    Jul 20 '14 at 18:21










  • $begingroup$
    Are negative primes allowed?
    $endgroup$
    – Mathmo123
    Jul 20 '14 at 18:23










  • $begingroup$
    Isn't a prime number positive by definition? Thanks Shaun I tried to make it better
    $endgroup$
    – user121591
    Jul 20 '14 at 18:25







  • 1




    $begingroup$
    I don't think your first line is correct. Suppose they are all greater than two. Then indeed all of the products of the form $pq-1$ are even. But that isn't a problem (even numbers can have odd prime divisors), and it doesn't immediately imply anything interesting.
    $endgroup$
    – Potato
    Jul 20 '14 at 18:35











  • $begingroup$
    By the way, I think you can piece a solution together from the comments here: nrich.maths.org/discus/messages/153904/149403.html?1281107322
    $endgroup$
    – Potato
    Jul 20 '14 at 18:40
















  • $begingroup$
    Here's a MathJax tutorial :)
    $endgroup$
    – Shaun
    Jul 20 '14 at 18:21










  • $begingroup$
    Are negative primes allowed?
    $endgroup$
    – Mathmo123
    Jul 20 '14 at 18:23










  • $begingroup$
    Isn't a prime number positive by definition? Thanks Shaun I tried to make it better
    $endgroup$
    – user121591
    Jul 20 '14 at 18:25







  • 1




    $begingroup$
    I don't think your first line is correct. Suppose they are all greater than two. Then indeed all of the products of the form $pq-1$ are even. But that isn't a problem (even numbers can have odd prime divisors), and it doesn't immediately imply anything interesting.
    $endgroup$
    – Potato
    Jul 20 '14 at 18:35











  • $begingroup$
    By the way, I think you can piece a solution together from the comments here: nrich.maths.org/discus/messages/153904/149403.html?1281107322
    $endgroup$
    – Potato
    Jul 20 '14 at 18:40















$begingroup$
Here's a MathJax tutorial :)
$endgroup$
– Shaun
Jul 20 '14 at 18:21




$begingroup$
Here's a MathJax tutorial :)
$endgroup$
– Shaun
Jul 20 '14 at 18:21












$begingroup$
Are negative primes allowed?
$endgroup$
– Mathmo123
Jul 20 '14 at 18:23




$begingroup$
Are negative primes allowed?
$endgroup$
– Mathmo123
Jul 20 '14 at 18:23












$begingroup$
Isn't a prime number positive by definition? Thanks Shaun I tried to make it better
$endgroup$
– user121591
Jul 20 '14 at 18:25





$begingroup$
Isn't a prime number positive by definition? Thanks Shaun I tried to make it better
$endgroup$
– user121591
Jul 20 '14 at 18:25





1




1




$begingroup$
I don't think your first line is correct. Suppose they are all greater than two. Then indeed all of the products of the form $pq-1$ are even. But that isn't a problem (even numbers can have odd prime divisors), and it doesn't immediately imply anything interesting.
$endgroup$
– Potato
Jul 20 '14 at 18:35





$begingroup$
I don't think your first line is correct. Suppose they are all greater than two. Then indeed all of the products of the form $pq-1$ are even. But that isn't a problem (even numbers can have odd prime divisors), and it doesn't immediately imply anything interesting.
$endgroup$
– Potato
Jul 20 '14 at 18:35













$begingroup$
By the way, I think you can piece a solution together from the comments here: nrich.maths.org/discus/messages/153904/149403.html?1281107322
$endgroup$
– Potato
Jul 20 '14 at 18:40




$begingroup$
By the way, I think you can piece a solution together from the comments here: nrich.maths.org/discus/messages/153904/149403.html?1281107322
$endgroup$
– Potato
Jul 20 '14 at 18:40










1 Answer
1






active

oldest

votes


















2












$begingroup$

The divisibility criteria ensure that $pqr$ divides
$$(pq-1)(pr-1)(qr-1)=p^2q^2r^2-p^2qr-pq^2r-pqr^2+pq+pr+qr-1,$$
and hence that $pqrmid pq+pr+qr-1$. Without loss of generality $pleq qleq r$ and so
$$3qr-1geq pq+pr+qr-1geq pqr,$$
which shows that $p<3$ and so $p=2$. Then $2qrmid2q+2r+qr-1$ and so $q$ and $r$ are odd, and moreover
$$2qrleq 2q+2r+qr-1,$$
or equivalently $(q-2)(r-2)leq3$. This shows that $q=3$ and $rin3,5$, and a quick check shows that $r=3$ does not work and that $r=5$ does, so $p,q,r=2,3,5$ and so $pqr=30$.






share|cite|improve this answer









$endgroup$













    Your Answer





    StackExchange.ifUsing("editor", function ()
    return StackExchange.using("mathjaxEditing", function ()
    StackExchange.MarkdownEditor.creationCallbacks.add(function (editor, postfix)
    StackExchange.mathjaxEditing.prepareWmdForMathJax(editor, postfix, [["$", "$"], ["\\(","\\)"]]);
    );
    );
    , "mathjax-editing");

    StackExchange.ready(function()
    var channelOptions =
    tags: "".split(" "),
    id: "69"
    ;
    initTagRenderer("".split(" "), "".split(" "), channelOptions);

    StackExchange.using("externalEditor", function()
    // Have to fire editor after snippets, if snippets enabled
    if (StackExchange.settings.snippets.snippetsEnabled)
    StackExchange.using("snippets", function()
    createEditor();
    );

    else
    createEditor();

    );

    function createEditor()
    StackExchange.prepareEditor(
    heartbeatType: 'answer',
    autoActivateHeartbeat: false,
    convertImagesToLinks: true,
    noModals: true,
    showLowRepImageUploadWarning: true,
    reputationToPostImages: 10,
    bindNavPrevention: true,
    postfix: "",
    imageUploader:
    brandingHtml: "Powered by u003ca class="icon-imgur-white" href="https://imgur.com/"u003eu003c/au003e",
    contentPolicyHtml: "User contributions licensed under u003ca href="https://creativecommons.org/licenses/by-sa/3.0/"u003ecc by-sa 3.0 with attribution requiredu003c/au003e u003ca href="https://stackoverflow.com/legal/content-policy"u003e(content policy)u003c/au003e",
    allowUrls: true
    ,
    noCode: true, onDemand: true,
    discardSelector: ".discard-answer"
    ,immediatelyShowMarkdownHelp:true
    );



    );













    draft saved

    draft discarded


















    StackExchange.ready(
    function ()
    StackExchange.openid.initPostLogin('.new-post-login', 'https%3a%2f%2fmath.stackexchange.com%2fquestions%2f872810%2fgiven-prime-numbers-p-q-and-r-pqr-%25e2%2588%2592-1-qrp-%25e2%2588%2592-1-and-rpq-%25e2%2588%2592-1-what%23new-answer', 'question_page');

    );

    Post as a guest















    Required, but never shown

























    1 Answer
    1






    active

    oldest

    votes








    1 Answer
    1






    active

    oldest

    votes









    active

    oldest

    votes






    active

    oldest

    votes









    2












    $begingroup$

    The divisibility criteria ensure that $pqr$ divides
    $$(pq-1)(pr-1)(qr-1)=p^2q^2r^2-p^2qr-pq^2r-pqr^2+pq+pr+qr-1,$$
    and hence that $pqrmid pq+pr+qr-1$. Without loss of generality $pleq qleq r$ and so
    $$3qr-1geq pq+pr+qr-1geq pqr,$$
    which shows that $p<3$ and so $p=2$. Then $2qrmid2q+2r+qr-1$ and so $q$ and $r$ are odd, and moreover
    $$2qrleq 2q+2r+qr-1,$$
    or equivalently $(q-2)(r-2)leq3$. This shows that $q=3$ and $rin3,5$, and a quick check shows that $r=3$ does not work and that $r=5$ does, so $p,q,r=2,3,5$ and so $pqr=30$.






    share|cite|improve this answer









    $endgroup$

















      2












      $begingroup$

      The divisibility criteria ensure that $pqr$ divides
      $$(pq-1)(pr-1)(qr-1)=p^2q^2r^2-p^2qr-pq^2r-pqr^2+pq+pr+qr-1,$$
      and hence that $pqrmid pq+pr+qr-1$. Without loss of generality $pleq qleq r$ and so
      $$3qr-1geq pq+pr+qr-1geq pqr,$$
      which shows that $p<3$ and so $p=2$. Then $2qrmid2q+2r+qr-1$ and so $q$ and $r$ are odd, and moreover
      $$2qrleq 2q+2r+qr-1,$$
      or equivalently $(q-2)(r-2)leq3$. This shows that $q=3$ and $rin3,5$, and a quick check shows that $r=3$ does not work and that $r=5$ does, so $p,q,r=2,3,5$ and so $pqr=30$.






      share|cite|improve this answer









      $endgroup$















        2












        2








        2





        $begingroup$

        The divisibility criteria ensure that $pqr$ divides
        $$(pq-1)(pr-1)(qr-1)=p^2q^2r^2-p^2qr-pq^2r-pqr^2+pq+pr+qr-1,$$
        and hence that $pqrmid pq+pr+qr-1$. Without loss of generality $pleq qleq r$ and so
        $$3qr-1geq pq+pr+qr-1geq pqr,$$
        which shows that $p<3$ and so $p=2$. Then $2qrmid2q+2r+qr-1$ and so $q$ and $r$ are odd, and moreover
        $$2qrleq 2q+2r+qr-1,$$
        or equivalently $(q-2)(r-2)leq3$. This shows that $q=3$ and $rin3,5$, and a quick check shows that $r=3$ does not work and that $r=5$ does, so $p,q,r=2,3,5$ and so $pqr=30$.






        share|cite|improve this answer









        $endgroup$



        The divisibility criteria ensure that $pqr$ divides
        $$(pq-1)(pr-1)(qr-1)=p^2q^2r^2-p^2qr-pq^2r-pqr^2+pq+pr+qr-1,$$
        and hence that $pqrmid pq+pr+qr-1$. Without loss of generality $pleq qleq r$ and so
        $$3qr-1geq pq+pr+qr-1geq pqr,$$
        which shows that $p<3$ and so $p=2$. Then $2qrmid2q+2r+qr-1$ and so $q$ and $r$ are odd, and moreover
        $$2qrleq 2q+2r+qr-1,$$
        or equivalently $(q-2)(r-2)leq3$. This shows that $q=3$ and $rin3,5$, and a quick check shows that $r=3$ does not work and that $r=5$ does, so $p,q,r=2,3,5$ and so $pqr=30$.







        share|cite|improve this answer












        share|cite|improve this answer



        share|cite|improve this answer










        answered Mar 29 at 22:24









        ServaesServaes

        30k342101




        30k342101



























            draft saved

            draft discarded
















































            Thanks for contributing an answer to Mathematics Stack Exchange!


            • Please be sure to answer the question. Provide details and share your research!

            But avoid


            • Asking for help, clarification, or responding to other answers.

            • Making statements based on opinion; back them up with references or personal experience.

            Use MathJax to format equations. MathJax reference.


            To learn more, see our tips on writing great answers.




            draft saved


            draft discarded














            StackExchange.ready(
            function ()
            StackExchange.openid.initPostLogin('.new-post-login', 'https%3a%2f%2fmath.stackexchange.com%2fquestions%2f872810%2fgiven-prime-numbers-p-q-and-r-pqr-%25e2%2588%2592-1-qrp-%25e2%2588%2592-1-and-rpq-%25e2%2588%2592-1-what%23new-answer', 'question_page');

            );

            Post as a guest















            Required, but never shown





















































            Required, but never shown














            Required, but never shown












            Required, but never shown







            Required, but never shown

































            Required, but never shown














            Required, but never shown












            Required, but never shown







            Required, but never shown







            Popular posts from this blog

            Triangular numbers and gcdProving sum of a set is $0 pmod n$ if $n$ is odd, or $fracn2 pmod n$ if $n$ is even?Is greatest common divisor of two numbers really their smallest linear combination?GCD, LCM RelationshipProve a set of nonnegative integers with greatest common divisor 1 and closed under addition has all but finite many nonnegative integers.all pairs of a and b in an equation containing gcdTriangular Numbers Modulo $k$ - Hit All Values?Understanding the Existence and Uniqueness of the GCDGCD and LCM with logical symbolsThe greatest common divisor of two positive integers less than 100 is equal to 3. Their least common multiple is twelve times one of the integers.Suppose that for all integers $x$, $x|a$ and $x|b$ if and only if $x|c$. Then $c = gcd(a,b)$Which is the gcd of 2 numbers which are multiplied and the result is 600000?

            Ingelân Ynhâld Etymology | Geografy | Skiednis | Polityk en bestjoer | Ekonomy | Demografy | Kultuer | Klimaat | Sjoch ek | Keppelings om utens | Boarnen, noaten en referinsjes Navigaasjemenuwww.gov.ukOffisjele webside fan it regear fan it Feriene KeninkrykOffisjele webside fan it Britske FerkearsburoNederlânsktalige ynformaasje fan it Britske FerkearsburoOffisjele webside fan English Heritage, de organisaasje dy't him ynset foar it behâld fan it Ingelske kultuergoedYnwennertallen fan alle Britske stêden út 'e folkstelling fan 2011Notes en References, op dizze sideEngland

            Հադիս Բովանդակություն Անվանում և նշանակություն | Դասակարգում | Աղբյուրներ | Նավարկման ցանկ